The mean for the normal hemoglobin control is 14.0 mg/dL. The standard deviation is 0.15 with an acceptable control range of +/-2 standard deviations (SD). What are the acceptable limits of the control? Please select the single best answer 13.8-14.2 13.4 14.6 13.6-14.4 13.7-14.3

Answers

Answer 1

The acceptable limits of the control is 13.7-14.3.Hemoglobin (Hb) is a red, iron-rich protein that allows red blood cells to transport oxygen throughout the body. Hemoglobin is responsible for the characteristic red color of blood and is responsible for the exchange of oxygen and carbon dioxide in the lungs.

The standard deviation (SD) is a statistical measure of the dispersion of a set of data values relative to its mean. A low standard deviation indicates that the data points are near to the mean, whereas a high standard deviation indicates that the data points are far from the mean. The standard deviation is expressed in the same units as the data points themselves. In the given problem, the mean for the normal hemoglobin control is 14.0 mg/dL and the standard deviation is 0.15. Since the acceptable control range is +/-2 standard deviations (SD), we can calculate the acceptable limits of control using the given formula below: Lower limit = Mean - 2(SD)Upper limit = Mean + 2(SD)Substitute the given values in the formula. Lower limit = 14 - 2(0.15)Upper limit = 14 + 2(0.15)Lower limit = 13.7Upper limit = 14.3Therefore, the acceptable limits of control are 13.7-14.3.Answer: 13.7-14.3

To know more about Hemoglobin, visit:

https://brainly.com/question/31239540

#SPJ11


Related Questions

Please help 20 points question

Please help 20 points question

Answers

Answer:

y>-2x-1 answer is b

-4x+y> 1 is c I believe

Step-by-step explanation:

The correct answer should be A

The number of pancakes y is proportional to the cups of pancake mix x that is used to make the pancake batter. The pancake batter will make 10 pancakes when 2 cups of pancake mix is used. Write an equation that represents the situation.

Answers

Answer:

y=5x

Step-by-step explanation:

10=m x 2

m=5

Brainliest?

Answer: For the first one, it’s y=5x and even though you didn’t put it, for the second question, it’s 30 dollars

Step-by-step explanation:

1. 10/2=5. Y=5x

2. 5*5 = $25.00

What is the mean of the data set?
8, 16, 9, 11

Answers

Answer:

The answer is 11

Step-by-step explanation:

Add all the numbers together

8+16+9+11=44

Then divide by 4 because there are only 4 numbers

so your answer would be 11

The density of glycerin is 20 g/cm3 at 20 0c. find the density of glycerin at 60 0c. the volume coefficient of glycerin is 5.1 x 10-4 0c-1.

Answers

The density of glycerin at 60°C is approximately 19.9592 g/cm³.

To find the density of glycerin at 60°C, we can use the volume expansion coefficient and the given density at 20°C.

The formula for volume expansion is:

ΔV = β * V₀ * ΔT

where:

ΔV is the change in volume,

β is the volume expansion coefficient,

V₀ is the initial volume, and

ΔT is the change in temperature.

In this case, we want to find the change in density, so we can rewrite the formula as:

Δρ = -β * ρ₀ * ΔT

where:

Δρ is the change in density,

β is the volume expansion coefficient,

ρ₀ is the initial density, and

ΔT is the change in temperature.

Given:

ρ₀ = 20 g/cm³ (density at 20°C)

β = 5.1 x 10⁻⁴ °C⁻¹ (volume expansion coefficient)

ΔT = 60°C - 20°C = 40°C (change in temperature)

Substituting the values into the formula, we have:

Δρ = - (5.1 x 10⁻⁴ °C⁻¹) * (20 g/cm³) * (40°C)

Calculating the expression:

Δρ = - (5.1 x 10⁻⁴) * (20) * (40) g/cm³

≈ - 0.0408 g/cm³

To find the density at 60°C, we add the change in density to the initial density:

ρ = ρ₀ + Δρ

= 20 g/cm³ + (-0.0408 g/cm³)

≈ 19.9592 g/cm³

Therefore, the density of glycerin at 60°C is approximately 19.9592 g/cm³.

To know more about volume expansion visit:

https://brainly.com/question/33519730

#SPJ11

estimate a polynomial regression using period, per squared, and dummy variables for feb-dec. do not remove any variables from the equation regardless of p values. do not add any variables.
2 decimal places, negative numbers use format -5 not (5) f=
Varibale Coeficient intercept = per = per2=
Feb =
Mar =
Apr =

Answers

The given task involves estimating a polynomial regression model using period, per squared, and dummy variables for the months of February to December.

The equation should include all variables without removing them based on their p-values, and no additional variables should be added. The desired format for coefficient values is two decimal places, and negative numbers should be displayed as "-5" instead of "(5)".

To estimate a polynomial regression model, we need to specify the equation that relates the dependent variable to the independent variables. In this case, the independent variables are period, per squared, and dummy variables for the months from February to December.

The equation for the polynomial regression model would look like this:

f = Intercept + Variable * period + Variable * per squared + Variable * Feb + Variable * Mar + Variable * Apr + ...

Each variable is multiplied by its corresponding independent variable. The intercept term represents the constant value in the equation. The coefficients for each variable determine the impact or contribution of that variable to the dependent variable.

To estimate the polynomial regression model, you need to provide the coefficient values for each variable. The desired format for the coefficients is two decimal places. For negative numbers, use the format "-5" instead of "(5)".

Please provide the coefficient values for the intercept, period, per squared, Feb, Mar, Apr, and any additional variables you have included in the model.

Learn more about squared here: https://brainly.com/question/14198272

#SPJ11

g(x) = 4x^2 – 16x +3

Answers

Answer:

https://brainly.com/question/18832765

Step-by-step explanation:

What am I solvinG for ?????

The three sides of a triangle are (x+5)cm ,(2x+4)cm and (2x-3)cm .
a.find the perimeter of the triangle
b. if x =10, find the perimeter of the traingle.

Answers

Hey ! there

Answer:

a. Perimeter = ( 5x + 6 ) cm

b. Perimeter when x is equal to 10 = 56 cm

Step-by-step explanation:

In this question we are given with three sides of a triangle that are as follows ,

( x + 5 ) cm

( 2x + 4 ) cm

( 2x - 3 ) cm

And we are asked to :

a. find perimeter of triangle.

b. find perimeter if x is equal to 10 .

We know that for finding perimeter of any shape we must have to add all the sides of the shape or ,

\( \qquad \: \underline{\boxed{\frak{Perimeter = Sum \: of \: all \: sides}}}\)

Solution : -

( a )

Now adding all the three sides of triangle to find the perimeter .

\( \hookrightarrow \quad \: ( x + 5 ) + ( 2 x + 4 ) + ( 2x - 3 ) \)

\( \hookrightarrow \quad \: \: x + 5 + 2x + 4 + 2x - 3\)

Combining like terms :

\( \hookrightarrow \quad \:x + 2x + 2x + 5 + 4 - 3\)

Now , solving :

\( \hookrightarrow \quad \: \red{\underline{ \boxed{ \frak{(5x + 6) \: cm}}}}\)

Therefore , perimeter of triangle is 5x + 6 centimetres .

( b )

Now , we are finding perimeter of triangle when value of x is 10 . So substituting value of x in given sides of triangle ,

First Side :

x + 5

10 + 5

15 cm

Second Side :

2x + 4

2 ( 10 ) + 4

20 + 4

24 cm

Third Side :

2x - 3

2 ( 10 ) - 3

20 - 3

17

So , all the sides are 15 cm , 24 cm and 17 cm .

Now , adding all these to find perimeter .

\( \hookrightarrow \qquad \: 15 + 24 + 17\)

\(\hookrightarrow \qquad \: 39 + 17\)

\(\hookrightarrow \qquad \: \red{\underline{\boxed{\frak{56 \: cm}}}}\)

Therefore , perimeter of triangle when value of x is 10 cm is 56 cm .

Alternative Solution : -

As above we have find the perimeter of triangle in term of x that is ( 5x + 6 ) cm. So we can put value of x as 10 in this to get perimeter. So ,

5x + 6

5 ( 10 ) + 6

50 + 6

56 cm

Therefore, perimeter of triangle is 56 cm .

#Keep Learning
Given ↓The three sides of the triangle : (x+5), (2x+4) and (2x-3)

To Find ↓ The perimeter of the triangleCalculations ↓

To find the perimeter of a shape, we should add all sides , like so :

x+5+2x+4+2x-3

Combine the x's :

x+2x+2x+5+4-3

x+4x+9-3

5x+6

Perimeter :

5x+6 cm

º º º º º º º º º º º º º º º º º º º º º º º º º º º º º º º º º º º º º º º º º º º º º

b.

Find the perimeter of the triangle if x = 10

Just write 10 instead of x :

5(10)+6

5(10) simplifies to 50 ↓

50+6

56

Perimeter of the triangle when x = 10 : 56 cm

hope helpful ~

Let f be the function given by f(x) 9x. If four subintervals of equal length are used, what is the value of the right Riemann sum approximation for (x) dx?

Answers

The value of the right Riemann sum approximation for integral ∫₀² f(x) dx is (c) 60.

The right Riemann sum approximation is obtained by dividing the interval [0, 2] into four subintervals of equal length and evaluating the function at the right endpoints of each subinterval. In this case, each subinterval has a length of (2-0)/4 = 0.5. The right endpoints of the subintervals are 0.5, 1.0, 1.5, and 2.0.

To calculate the right Riemann sum, we evaluate the function at these right endpoints and sum up the values multiplied by the subinterval length.

f(0.5) = \(9^{0.5\) = 3

f(1) = 9¹ = 9

f(1.5) = \(9^{1.5\) = 27

f(2) = 9² = 27

The right Riemann sum is then

= (0.5 * f(0.5)) + (0.5 * f(1.0)) + (0.5 * f(1.5)) + (0.5 * f(2.0))

= 0.5 * (3 + 9 + 27 + 81)

= 60.

Therefore, the value of the right Riemann sum approximation for ∫2 to 0 f(x) dx is 60, which corresponds to option (c).

Learn more about Riemann sum here

https://brainly.com/question/30404402

#SPJ4

Given question is incomplete, the complete question is below

let f be the function given by f(x)= 9ˣ, if four subintervals of equal length are used, what is the value of the right riemann sum approximation for∫₀² f(x) dx. 20b. 40c. 60d. 80

Rectangles ABCD and WXYZ are similar, but not congruent. Which sequence of transformations could map ABCD to WXYZ?
Transformations
rotation, dilation
rotation, reflection
translation, dilation
translation, reflection

Answers

For all the given sequence of transformations, Rectangles ABCD and WXYZ will be similar.

What is the difference between upward, downward, leftwards and rightwards translation?

Upward translation = [k] units -When the upward translation take's place then, each point will have its y - coordinate shifted upwards by +k units.Downward translation = [k] units - When the downward translation will take place then, each point will have its y - coordinate shifted downwards by -k units.Leftward Translation = [k] units - When the leftward translation will take place then, each point will have its x - coordinate shifted by -k units to the left.Rightward Translation = [k] units - When the rightward translation will take place then, each point will have its x - coordinate shifted by +k units to the right.

It is given that Rectangles ABCD and WXYZ are similar, but not congruent.

For all the transformations given represents the similar rectangles. Dilations, rotations, translations and reflections all lead to similar figures.

Therefore, for all the given sequence of transformations, Rectangles ABCD and WXYZ will be similar.

To solve more questions on Graph translations, visit the link below -

brainly.com/question/12802031

#SPJ9

Write an equation of the line that is perpendicular to line f and passes through point Q. Show or explain how you got your answer

Write an equation of the line that is perpendicular to line f and passes through point Q. Show or explain

Answers

y + 1 = 2x is the equation of the line that passes through the points (0,-1) and (0.5, 0) and also passes through the line f in the graph.

what is equation ?

An equation is a mathematical statement that asserts that two expressions are equal. It is typically written using an equal sign (=) between the two expressions. An equation can contain variables, which are symbols that represent unknown values.

In the given question,

To write an equation of the line that passes through the points (0,-1) and (0.5, 0), we first need to find the slope of the line using the slope formula:

slope = (y₂ - y₁) / (x₂ - x₁)

Substituting the coordinates of the two points, we get:

slope = (0 - (-1)) / (0.5 - 0)

slope = 1 / 0.5

slope = 2

Now that we know the slope of the line, we can use the point-slope form of the equation of a line to write the equation:

y - y1 = m(x - x₁)

Substituting the coordinates of one of the points and the slope, we get:

y - (-1) = 2(x - 0)

y + 1 = 2x

This is the equation of the line that passes through the points (0,-1) and (0.5, 0) in the graph.

To know more about equation , visit:

brainly.com/question/29657983

#SPJ1

how to solve the attachment question?

how to solve the attachment question?

Answers

Answer:

y = 100 - 0.08^ x

Step-by-step explanation:

can someone help me

can someone help me

Answers

V = 85
All angles in a triangle equal to 180
can someone help me

The school play sold $897 in tickets one night. The number of $8 adult tickets was 15 more than twice the number of $5 child tickets. How many of each ticket were sold?
Input your answer on the following lines.

Answers

Answer:Child ticket sold = 30

Adult ticket sold = 50

Total ticket sold = 80

Step-by-step explanation:

∠1 ​ and ∠2 are complementary. m∠1=x°m∠2=(3x 30)° what is the value of x? select from the drop-down menu to correctly answer the question.

Answers

The value of x is 15

If two angles sum to 90 degrees, they are said to be complimentary angles. In other terms, a right angle is created when two complementary angles are combined (90 degrees). If the sum of angles 1 and 2 equals 90 degrees (i.e., angle 1 plus angle 2 = 90°), then the angles are complementary and are referred to as one another's complements.

∠1 and ∠2 are complementary angles i.e the sum of ∠1 and ∠2 is 90° .

Thus

∠1 + ∠2 = 90°

x° + (3x+30)° = 90°

x + 3x + 30 = 90

4x = 90-30

4x = 60

x = 15

Therefore the value of x is 15.

Learn more about angles:

https://brainly.com/question/28451077

#SPJ4

Write the coordinates of the vertices after a rotation 90° counterclockwise around the origin.

Answers

When rotating a point 90 degrees counterclockwise about the origin our point A(x,y) becomes A'(-y,x). In other words, switch x and y and make y negative

What is the explanation?

U(2, 7) → U'(-7, 2)

V(10, 7) → V'(-7, 10)

W(2, 3) → W'(-3, 2)

Step-by-step explanation:

From the picture attached,

Coordinates of the vertices of the given triangle,

U → (2, 7)

V → (10. 7)

W → (2, 3)

Since rule for the coordinates of a point P(x, y) after a rotation of 90° counterclockwise about the origin is,

to more about rotation 90° refer to:

https://brainly.com/question/21289953

#SPJ13

Jada and Elena learned that 8% of students have asthma. They want to know the probability that in a team of 4 students, at least one of them has asthma. To simulate this, they put 25 slips of paper in a bag. Two of the slips say "asthma." Next, they take four papers out of the bag and record whether at least one of them says "asthma." They repeat this process 15 times. Jada says they could improve the accuracy of their simulation by using 100 slips of paper and marking 8 of them. Elena says they could improve the accuracy of their simulation by conducting 30 trials instead of 15. Do you agree with either of them?

Answers

Answer:

By conducting 30 trials instead of 15, they would improve the accuracy of their simulation, therefore, Elena is correct

Step-by-step explanation:

The parameters given are;

Jada and Elena are trying to estimate the probability that 1 in 4 students selected has asthma where the asthma prevalence rate = 8%

The experiment uses 25 slips to represent 25 students and they labelled 2 of the strips as asthma to indicate the expected number of asthma in 25 students which is 2/25 = 0.08. That is the number of asthma in 25 students is 8% of 25 = 0.08 × 25 = 2

From the 25 slips, they randomly select groups of 4 which represent the team of four students to see if any of the 4 slips is one of the two slips labelled as asthma slips

Therefore, to improve the accuracy of the estimate of the probability they should increase the number of trials from 15 to 30, hence Elena is correct.

a certain test preparation course is designed to help students improve their scores on the lsat exam. a mock exam is given at the beginning and end of the course to determine the effectiveness of the course. the following measurements are the net change in 5 students' scores on the exam after completing the course: 13,19,22,15,29 using these data, construct a 80% confidence interval for the average net change in a student's score after completing the course. assume the population is approximately normal. step 1 of 4 : calculate the sample mean for the given sample data. round your answer to one decimal place.

Answers

The sample mean for the given sample data is 19.6

using formula Mean = (Σxi)/n

Where Σ tells us to add,

xi refers to all the X-values

and n stands for the number of items in the data set.

What is a sample mean?

The sample mean of a data set is used by statisticians to estimate the standard of normality within a given population, as well as to calculate the variance, deviation, and standard error within a data set.

The average of a group of data is called a sample mean. A data set's central tendency, standard deviation, and variance can all be determined using the sample mean. A population average can be determined using the sample mean, among other things.

xi = 13,19,22,15,29

Total no. of students (n) = 5

Sample Mean = ( Σ xi ) / n

⇒ Sample Mean = (13+19+22+15+29)/5

⇒ Sample Mean = 98/5

⇒ Sample Mean = 19.6

The sample mean for the given sample data is 19.6

To learn more about sample mean visit:

brainly.com/question/30023845

#SPJ4

Which of the following ordered pairs represent points that lie on a horizontal line?

Which of the following ordered pairs represent points that lie on a horizontal line?

Answers

The ordered pairings (-2, 3), and (-5, 3), respectively, depict points along a horizontal line.

What is horizontal line?

Let's go over some information on the horizontal line.

If the y-coordinates of every point along the line are equal, the line is horizontal.

The horizontal line's slope is zero and it is perpendicular to the x-axis.

Because the y-coordinates of every point on the line equal b, the equation of the horizontal line through point (a, b) is y = b.

Find the points with the same y-coordinates in order to identify the ordered pairs that represent points along a horizontal line.

Response A:

The paired numbers are (-2, 3), and (-5 , 3)

X is located between -2 and -5.

3 and 3 are the y-coordinates.

The two ordered pairs' y-coordinates are equivalent.

Points (-2, 3) and (-5, 3) are adjacent on a horizontal axis.

To learn more about  horizontal line refer to:

https://brainly.com/question/14197570

#SPJ1

Let the long-run profit function for a representative firm is given by π i

=p 2
−2p−399, where p is the price of computer. The inverse market demand for computer is given by p=39−0.009q, where q is unit of computers. Suppose technology for producing computers is identical for all firms and all firms face identical input prices. (a) Find the firm's output supply function. (b) Find the market-equilibrium price and the equilibrium number of firms. (c) Find the number of computers sold by each firm in the long run.

Answers

(a) The firm's output supply function is given by q = (p + 199) / 2.

(b) The market-equilibrium price is $32.56, and the equilibrium number of firms is 10.

(c) Each firm sells 70 computers in the long run.

To find the firm's output supply function, we need to maximize the firm's profit function, which is given by π = p^2 - 2p - 399. In the long run, firms will produce where marginal cost equals marginal revenue. Marginal revenue can be obtained by differentiating the inverse market demand function with respect to q, and marginal cost is equal to the derivative of the profit function with respect to q. Equating the two, we get:

(39 - 0.009q) = (2q - 2) / q

Simplifying the equation, we find:

q = (p + 199) / 2

This represents the firm's output supply function.

To find the market-equilibrium price and the equilibrium number of firms, we need to find the intersection point of the market demand and supply. Substituting the output supply function into the inverse market demand function, we have:

p = 39 - 0.009((p + 199) / 2)

Simplifying and solving for p, we get:

p ≈ $32.56

Substituting this price back into the output supply function, we find:

q = (32.56 + 199) / 2 ≈ 115.78

Given that each firm produces 70 computers in the long run, we can calculate the equilibrium number of firms:

Number of firms = q / 70 ≈ 10

Since each firm sells 70 computers in the long run, and there are 10 firms, the total number of computers sold by each firm is:

70 * 10 = 700

Learn more about  Equilibrium

brainly.com/question/30694482

#SPJ11

Select all that are measures of angles coterminal with a 145° angle. ""575° ""215° ""145° ""35° 215° 415° 505° 865°

Answers

The angles coterminal with a 145° angle are 215°, 505°, and 695° (or -145°, -505°, and -695°). So, the correct option are E, G and H.

The measures of angles coterminal with a 145° angle are

215° = (145° + 360°)

505° = (145° + 360° + 360°)

865° = (145° + 360° + 360° + 360°)

The angles that are coterminal with a 145° angle are those that can be obtained by adding or subtracting a multiple of 360°. Therefore, the angles that are coterminal with 145° are 505° and 865°. The other angles listed are not coterminal with 145°.

Therefore, the correct answer are 215°, 505°, 865° and options are E, G and H.

To know more about Angles:

brainly.com/question/28451077

#SPJ4

Solve the equation.

p−3=−4
p=

Answers

The value of p in the given equation is -1.

Given is an equation p-3 = -4, we need to find the value of p,

So,

p-3 = -4

p = -4+3

p = -1

Hence, the value of p in the given equation is -1.

Learn more about equations, click;

https://brainly.com/question/29657983

#SPJ1

$80 is 25% of what amount?

Answers

It’s actually 320 because 25% times 4 equals to 100 so just multiply 80x4 and it equals 320. Hope this helps

Find the 14th term of the geometric sequence 5 , − 10 , 20 ,

Answers

Answer:

-40960

Step-by-step explanation:

The formula for geometrc sequence is:

\(\displaystyle{a_n = a_1r^{n-1}}\)

Where r represents common ratio. In this sequence, our common ratio is -2 as -10/5 = -2 as well as 20/-10 = -2.

\(a_1\) represents the first term which is 5. Therefore, by substitution, we have:

\(\displaystyle{a_n = 5(-2)^{n-1}}\)

Since we want to find the 14th term, substitute n = 14. Thus:

\(\displaystyle{a_{14} = 5(-2)^{14-1}}\\\\\displaystyle{a_{14}=5(-2)^{13}}\\\\\displaystyle{a_{14} = 5(-8192)}\\\\\displaystyle{a_{14}=-40960}\)

Therefore, the 14th term is -40960.

write in simplest form7 × 3/5 =

Answers

The simplest form is:

\(7\cdot\frac{3}{5}=\frac{7\cdot3}{5}=\frac{21}{5}\)

Help me ASAP ! Plssss

Help me ASAP ! Plssss

Answers

Answer:

ok

1 b

2c

3a

4f

5d

6g

7e

9g

10i

find three numbers. their sum is 3 and their difference is 21

Answers

Answer:

Step-by-step explanation:

Let's call the three numbers we're trying to find "a", "b", and "c". We know that their sum is 3, so we can write:

a + b + c = 3

We also know that their difference is 21. However, we don't know if that difference is positive or negative, so we'll write two equations:

a - b - c = 21

b - a - c = 21

Now we have a system of three equations with three unknowns. We can solve for one variable in terms of the other two, and then substitute that expression into the other two equations to get a system of two equations with two unknowns. From there, we can solve for one variable and use that to find the other two.

Here's one way to solve the system:

a + b + c = 3

a - b - c = 21

b - a - c = 21

Adding the second and third equations, we get:

a - c = 42

Substituting this into the first equation, we get:

b + 42 = 3

b = -39

Substituting b and a - c into the second equation, we get:

a - (-39) - (a - c) = 21

a + 39 - a + c = 21

c = -18

Now we can find a by adding b and c to 3:

a = 3 - b - c

a = 3 - (-39) - (-18)

a = 24

So the three numbers are 24, -39, and -18, and their sum is 3 and their difference is 21.

Parallelogram ABCD is rotated to create image A'B'C'D'.

Answers

The transformation rule that describes the rotation from the original parallelogram ABCD to the image parallelogram A'B'C'D' is (x, y) → (–y, x).

The rule that describes the transformation from the original parallelogram ABCD to the image parallelogram A'B'C'D' is (x, y) → (–y, x).

To understand this, let's apply the transformation rule to each vertex of the original parallelogram ABCD:

Point A (2, 5) becomes A' (-5, 2).

Point B (5, 4) becomes B' (-4, 5).

Point C (5, 2) becomes C' (-2, 5).

Point D (2, 3) becomes D' (-3, 2).

By applying the transformation rule, we observe that the x-coordinate of each point becomes the negative of the original y-coordinate, and the y-coordinate becomes the original x-coordinate.

This transformation is a 90-degree counterclockwise rotation about the origin (0, 0) on the coordinate plane. The image parallelogram A'B'C'D' is obtained by rotating the original parallelogram ABCD by 90 degrees counterclockwise.

Visually, this transformation can be seen as the original parallelogram being rotated around the origin, where the x-axis becomes the y-axis, and the y-axis becomes the negative x-axis.

Therefore, the transformation rule that describes the rotation from the original parallelogram ABCD to the image parallelogram A'B'C'D' is (x, y) → (–y, x).

For more such questions on parallelogram visit:

https://brainly.com/question/970600

#SPJ8

Parallelogram ABCD is rotated to create image A'B'C'D'.

PLEASE HELP QUICK AHHHH <333

PLEASE HELP QUICK AHHHH &lt;333

Answers

Answer:

=> ( -6,-10 )

Step-by-step explanation:

Given :

x = -6

-7x + 6y = -18 ... i)

Putting the value of x in equation ...i)

=> -7 (-6) + 6y = -18

=> 42 + 6y = -18

=> 6y = -18 -42

=> 6y = - 60

=> y = -60/6

=> y = -10

The value of y = -10 and x = -6

Answer:

x= 4, y=6

Step-by-step explanation:

9x-9y = -18

9x=-18+9y

x=-2+y

-6(-2+y) + 9y = 6

-12-6y+9y=6

3y=18

y=6

9x -9(6)=-18

9x-54=-16

9x=36

x=4

Amy bought 3 bags of 8 party favors She divided the party favors evenly amongst 6 guests. How many favors did each guest get?

Answers

Answer:

4 party favors per guest

Step-by-step explanation:

3 bags multiplied by 8 party favors is equal to 24 party favors

then divide 24 party favors by 6 people to get 4 party favors per guest

In math class, Sam correctly multiplied 3. 625×10-8 by 500, then reported his product in scientific notation. What was the exponent in Sam’s product? A -16 B -10 C -7 D -5

Answers

The exponent in Sam’s product is -5.

What does a math exponent mean?

When a number is multiplied by itself, it is said to have an exponent. For instance, 2 to the third (written as 23) indicates that 2 x 2 x 2 = 8. 23 and 2 x 3 = 6 are not equivalent. A number raised to the power of one is itself, so keep that in mind.

Let us first multiply 500 with 3. 625×10-8

3. 625×10-8  × 500 = 1812.5 × 10⁻⁸

Then Sam has reported this answer in scientific notation

The format for writing a number in scientific notation is

(first digit of the number) followed by (the decimal point) and then (all the rest of the digits of the number), times (10 to an appropriate power).

In 1812.5 × 10⁻⁸  we move the decimal 3 times to left side

Since the decimal is moved left side, we multiply by 10³

1812.5 × 10⁻⁸ = 1812.5 × 10⁻⁸ × 10³

1812.5 × 10⁻⁸  = 1812.5 × 10⁻⁵

Exponent is a quantity representing the power to which a given number or expression is to be raised

1812.5 × 10⁻⁵ exponent is -5

Learn more about exponent

brainly.com/question/5497425

#SPJ4

Other Questions
On a school field trip, the number of students (y) is always proportional to the number of adults (x). In one group there are96 students and 8 adults. What is the constant of proportionality between this relationship? 1. Are you given enough information to prove that these two triangles are congruent? If so,what reason would you give? The difference of a number n and 1 squared is 25 Emily has pledged to start a healthy lifestyle and so she started walking to keep fit. On Monday, she walked 2/4 of a mile, and on Tuesday she walked 5/6 of a mile. How far did she walk in her first two days of exercise? Quickkkkkk A wave has an amplitude of 0.0800 mand is moving 7.33 m/s. One oscillatorin the wave takes 0.230 s to go fromone crest to the next crest. Find thewavelength of the wave.(Unit = m) How do I figure this out?? I dont get it _____ refers to non-financial companies spreading out its activities in different currency zones in order to offset the currency losses in certain regions through gains in other regions. Your client's child will be attending college in 5 years and she asks you how much she will need to set aside today to pay the first year's tuition and fees. She estimates that if her child attends her alma mater the tuition and fees will be $45,882. Assuming she can earn 7.84 percent on the money she invests for this purpose, compounded annually, how muchwould she need to invest today to meet her child's first year college need? Round the answer to two decimal places Describe the steps you would take to solve the equation 10.4x - 12.5 = -3.8. Do not solvethe equationStep 1:Step 2:Help haven20 min to get this Donne (USE THE CIRCLE GRAPH BELOW.)Magazine sales total $25,000.Boating 10%Sports 35%Other 5%Adult 20%Teen 30%How much more is made on Sports magazines than Teen magazines?A. $5B. $12.50c. $1.25D. $1250E. $12,500 Explain the House of Burgesses and why it is important to US History Today? 2. Define the law of diminishing returns. Why is this law so important within the field of managerial economics? 3. Explain returns to scale. Introduce and define the types of returns to scale within What is answer to this? replacing an employee who leaves a company may cost anywhere from of the annual salary of the departing employee. group of answer choices 80% to 125% 90% to 200% 100% to 150% 70% to 100% what is a commonly accepted set of assumptions What is the slope intercept form here!!! What changes are most likely to occur as a result of civil wars? Select all that apply. population movements destruction of roads increased economic activities destruction of buildings eleven bands are to perform at a weekend festival. how many different ways are there to schedule their appearances? After peripheral nerve damage, pharmacological blockage of which cell type would prevent repair? Kyla climbed to a height of 10 ft from the bottom of a rock climbing wall. She then climbed up an additional 5 ft. Kyla must climb down how many feet to return to the bottom of the climbing wall?